Re: GA / Baricentro de um Triângulo

2001-11-13 Por tôpico Bruno Fernandes Cerqueira Leite

Você pode achar a equação de uma mediana e lembrar-se que o baricentro
divide a mediana na razão 2:1. Acho que aí fica fácil.

Se vc usar vetores aí fica completamente óbvio!!

Espero ter ajudado,

Bruno Leite

At 18:55 13/11/01 -0200, you wrote:
> Olá...
>
> Estava quebrando a cabeça num problema do ITA 
>(http://www.exatas.f2s.com/matematica/ga005.html) quando achei a solução 
>usando a 'fórmula' do baricentro: G((xa + xb + xc)/ 3, (ya + yb + yc)/3) do 
>triângulo.
> Depois disso ficou bem fácil o exercício. mas fiquei me 
>perguntando aqui, de onde que essa fórmula vem. Procurei em vários livros 
>mas ela é sempre 'empurrada' e nunca demonstrada ou provada.
> Comecei a esboçar uma demonstração mas os cálculos ficaram muito 
>monstruosos. Parti dum triângulo ABC, sendo A(x1,y1), B(x2,y2) e C(x3,y3). 
>M sendo o ponto médio de AC, N ponto médio de AB e P o ponto médio de BC. A 
>partir disso achei a equação geral de duas medianas (primeiro calculando o 
>coefiente angular, a partir de /\y//\x, e dai jogando na formula da equacao 
>geral da reta), ambas gigantescas... ie.:
>EQG de BM: y(x1 + x3 - 2x2) - y2(x1 + x3 - 2x2) = x(y1 + y2 - 2y3) - x2(y1 
>+ y3 - 2y2)
>EQG de CN: y(x1 + x2 - 2x3) - y3(x1 + x2 - 2x3) = x(y1 + y2 - 2y3) - x3(y1 
>+ y2 - 2y3)
> A partir disso tentei trabalhar com esses dois 'monstros' , 
>isolando x numa e inserindo na outra, mas não fiz muitos progressos.. Não 
>há alguma outra maneira de demonstrar que o Baricentro de um triângulo 
>sempre corresponde a média simples de x e y?
>
>grato pela atenção..
>
>
>
>
>
>"Against stupidity, the Gods themselves contend in vain",
> Friedrich von Schiller's
>-
>[]'s
>{O-Grande-Mentecapto}
>[EMAIL PROTECTED]
>
>
>




Re: biografia (fwd)

2001-10-30 Por tôpico Bruno Fernandes Cerqueira Leite

At 19:09 18/10/01 -0300, you wrote:
>Ola a todos,
>
>Apareceu um problema na aula de cálculo I que eu nao conssigo fazer de nenhum
>jeito tentei de tudo, com certeza algo de errado eu fiz, por favor da uma
>mão.
>
>A reta y=5x+11 intercepta a parábola y=x^2 nos pontos A e B. Encontre o ponto
>P sobre o arco OAB da parábola que maximize a área do triangulo PAB. (O é a
>origem do plano cartesiano por onde x^2 passa)
>
>Fernando Romagnoli
>

Oi Fernando

Para maximizar a area de PAB vc tem que pegar P tal que a altura PH (H em
AB) seja máxima.

Vá traçando todas as retas paralelas a AB, isto é de coeficiente angular 5.
Vc não conseguirá aumentar a area de PAB quando P for o ponto de tangencia
entre uma reta de coeficiente 5 e a parábola, isto é, quando f'(x)=5. Logo
x_P=2,5 e y_P=(2,5)^2=6,25

Bruno Leite




Re: RES: Equação funcional

2001-10-27 Por tôpico Bruno Fernandes Cerqueira Leite

At 22:26 26/10/01 -0200, you wrote:
>Ei Eric!!
>
>F(x) = c num satisfaz a equação não viu...

Acho que ele quis dizer f(x)=x+c..

Bruno


>Se não ela seria constante e x +2c = c ???
>
>Nao entendi...
>
>flw
>Einstein
>
>-Mensagem original-
>De: [EMAIL PROTECTED] [mailto:[EMAIL PROTECTED]]Em
>nome de Nicolau C. Saldanha
>Enviada em: sexta-feira, 26 de outubro de 2001 11:59
>Para: [EMAIL PROTECTED]
>Assunto: Re: Equação funcional
>
>
>On Fri, Oct 26, 2001 at 11:28:25AM -0200, Eric Campos Bastos Guedes wrote:
>> Saudacoes a todos
>>
>> tenho uma duvida que eh a seguinte:
>>
>> quais as funcoes f:R->R que satisfazem f(f(x))=x+2c.  Eh claro que f(x)=c
>eh
>> uma dessas funcoes, mas existirao outras?  E como fica o problema se
>supomos
>> f continua ou monotona?  Hah alguma diferenca se f estiver definida em
>> apenas um intervalo da reta?
>
>Existem outras. Tome c = 1 e
>
>f(x) = x + 1/3 + {x},  0 <= {x} < 1/3
>f(x) = x + 5/6 - {x}/2,  1/3 <= {x} < 1
>
>onde {x} é a parte fracionária de x, i.e.,
>
>0 <= {x} < 1, x - {x} inteiro.
>
>[]s, N.
>
>




Re: Unicamp: Ensino Medio?!

2001-10-24 Por tôpico Bruno Fernandes Cerqueira Leite

At 11:16 24/10/01 -0300, you wrote:
>Legal essa questao. Fica facil se vc puder usar o teorema que diz que
dadas duas matrizes quadradas X,Y, se tem det(X.Y)=detX.detY.
>
>Nesse caso, basta inserir (n-m) colunas nulas a direita de A (criando a
matriz A´) para que esta vire quadrada, e inserir (n-m) linhas zeros abaixo
de B (criando B´).
>
>Da propria definicao do produto de matrizes, esses termos acrescentados
nao afetam em nada na multiplicacao das matrizes, i.e, 
>A.B = A´.B´. E pelo teorema enunciado la em cima, como tanto a matriz A
como a B tem pelo menos 1 (ja que n-m>=1) filas nulas, tem-se 
>det(AB)=det(A´B´)=0.0=0
>
>O problema eh que aquele teorema inicial nao costuma ser demonstrado no
ensino medio (embora seja sempre enunciado). Mas ateh ai tudo bem, pq a
regra de Laplace (expansao por cofatores) para o calculo do determinante
tmb nao costuma ser deduzida (as vezes ela eh dada como definicao, mas ai
acho que fica um pouco complicado mostrar que o det independe da fila onde
a expansao sera feita)! 
>
>Gostaria inclusive que o pessoal da lista comentasse sobre isso. Na
opiniao de voces, qual eh a maneira ideal de se abordar a teoria de
determinantes no ensino medio?
>
>Abracos,
>Marcio

Eu acho bem discutível o fato de determinantes ser matéria de 2º grau. Acho
que outras coisas mais interessantes e mais bonitas (como vetores, produto
vetorial, bases, etc) poderiam ser dadas no lugar.

Acho que na maioria dos colégios eles gastam 10 aulas com matrizes, soma de
matrizes, produto, inversa (raramente falam sobre a motivação para definir
inversa de uma matriz daquele jeito), transposta, adjunta. Mais 5 aulas e
ensinam a calcular determinantes, linhas nulas, Laplace, etc (e Laplace é
usado como definição, novamente sem motivação alguma)

Aí vemos a regra de Cramer, que é _A_ aplicação de determinantes no ensino
médio, e , por último, vemos que podemos esquecer tudo isso pq
escalonamento é muito melhor que Cramer. (e escalonamento é muito mais
simples para se ensinar!)

Não estou dizendo que determinantes não são importantes, mas acho que há
outros assuntos mais importantes para um aluno do ensino médio.

Bruno Leite


>-- Mensagem Original --
>De: "Alexandre F. Terezan" <[EMAIL PROTECTED]>
>Para: OBM <[EMAIL PROTECTED]>
>Enviar: 02:59 AM
>Assunto: Unicamp
>
>Uma questao da Unicamp:
> 
>Dada uma matriz A{n x m}  e uma matriz B{m x n}, onde 
>n>m.
> 
>Prove que det (A * B) = 0.
>
>
>
>
>
>




Re: OBM-u

2001-10-23 Por tôpico Bruno Fernandes Cerqueira Leite

At 15:22 23/10/01 -0200, you wrote:
>Ai vai o mail do Marcio:
>
>Bruno
>
>At 09:45 23/10/01 -0300, you wrote:
>>A minha solucao pra 5 foi assim:
>>A primeira parte da letra (a) era bem facil. A segunda eu demorei um pouco
>mais.. Primeiro troquei cosx por [2sen^(x/2) - 1] pra completar quadrados e
>separo em duas integrais, que depois de alguma conta, ficam iguais a
>2I[u].. (nao lembro exatamente como, mas eu tive que separar de 0 a Pi/2 e
>depois de Pi/2 ateh Pi).
>>Obs: Para quem nao viu a prova, todas as integrais abaixo estao sendo
>feitas em x:
>>
>>Na letra (b), minha ideia foi primeiro ver que:
>>I[u] = (1/2)I[u^2] = (1/4)I[u^4] = ... = (1/t)I[u^t] sempre que t eh
>potencia de dois.
>>1o caso: 0>Como cos <= 1, vc tem 1+2ucosx+u^2<=(1+2u+u^2)=(1+u)^2
>>Tirando log (crescente) e ja integrando, vc sempre tem:
>>|I[u]|=|I[-u]|<=|Int[log(1+2u+u^2)dx]|<=Int[|log(1+u)^2|dx]=2Pi*log|1+u|
>>Em particular, quando t eh uma potencia de 2, vc tem:
>>|I[u]|=|I[u^t]/t |<= 2Pi*log|1+u^t|/t .
>>Como 0(na verdade, qdo k vai pra infinito) para concluir que 
>>I[u]=0  nesse caso.
>>Usando que I[u]=I[-u], vc conclui que I[u]=0 se |u|<= 1(em 1 eh obvio)
>>
>>Por outro lado, se |z|>1, trocando u por 1/u na definicao de I vem: 
>>I(1/u)=Int[log(1-2cosx/u +1/u^2)] = 
>>Int[log(u^2 - 2ucosx + 1)] - Int[2log|u|]
>>Fazendo z = 1/u, temos que se |z|>1, entao |u|<1 e portanto:
>>I(z) = 0 -2Pilog|u| = 2Pi*log|z|, o q conclui o problema..


Uau, muito bonita esta solução! Só não entendi pq I(u) =I(-u).
(na verdade tb não sei pq vc trocou cosx por [2sen^(x/2) - 1], mas acho que
vc quis dizer [2cos^2(x/2) - 1]...)

Bruno






Re: OBM-u

2001-10-23 Por tôpico Bruno Fernandes Cerqueira Leite

At 13:45 23/10/01 -0200, you wrote:
>On Tue, Oct 23, 2001 at 09:01:28AM -0200, Bruno Fernandes Cerqueira Leite
wrote:
>> At 00:30 23/10/01 -0200, you wrote:
>> >Oi Bruno! Td bom? Tb achei a prova legal.. Qto ao resultado, acho que
>> >fiz a 1 e a 5, nao completei direito a 2 pq nao lembrava exatamente do
>> >enunciado (ou prova) de um teorema que tinha na Eureka 3 (no artigo de
>> >fracoes continuas) que me ajudaria muito. Na 4, que eu achei uma
questao bem
>> >interessante, eu tmb
>> >escrevi.
>> 
>> Podia usar o teorema da equidistribuição de {an} (a irracional, n natural)
>> mod 1 na questão 2?
>> Acho que se pudesse usar a questão ficaria quase trivial! (eu, por via das
>> dúvidas, não usei)
>> 
>> O teorema acima diz o seguinte (informal): a probabilidade de vc ter
>> x<{an}> ( onde {x}=x-[x] é a parte fracionária de x.) Isso mostra que a sequência
>> {an} é equidistribuida em [0,1).
>
>Claro que este teorema pode ser usado mas não acho que a questão fique
>tão trivial assim com este teorema. Lembrando, a questão é:
>
>Seja (epsilon) um número real positivo arbitrário.
>Com centro em todos os pontos do plano com coordenadas inteiras,
>traça-se um círculo de raio (epsilon).
>Prove que toda reta passando pela origem
>intercepta uma infinidade desses círculos.
>
>[]s, N.
>
Basta provarmos que {an}infinito, e acabou!

(está errado?)

Bruno




Re: OBM-u

2001-10-23 Por tôpico Bruno Fernandes Cerqueira Leite

Ai vai o mail do Marcio:

Bruno

At 09:45 23/10/01 -0300, you wrote:
>A minha solucao pra 5 foi assim:
>A primeira parte da letra (a) era bem facil. A segunda eu demorei um pouco
mais.. Primeiro troquei cosx por [2sen^(x/2) - 1] pra completar quadrados e
separo em duas integrais, que depois de alguma conta, ficam iguais a
2I[u].. (nao lembro exatamente como, mas eu tive que separar de 0 a Pi/2 e
depois de Pi/2 ateh Pi).
>Obs: Para quem nao viu a prova, todas as integrais abaixo estao sendo
feitas em x:
>
>Na letra (b), minha ideia foi primeiro ver que:
>I[u] = (1/2)I[u^2] = (1/4)I[u^4] = ... = (1/t)I[u^t] sempre que t eh
potencia de dois.
>1o caso: 0Como cos <= 1, vc tem 1+2ucosx+u^2<=(1+2u+u^2)=(1+u)^2
>Tirando log (crescente) e ja integrando, vc sempre tem:
>|I[u]|=|I[-u]|<=|Int[log(1+2u+u^2)dx]|<=Int[|log(1+u)^2|dx]=2Pi*log|1+u|
>Em particular, quando t eh uma potencia de 2, vc tem:
>|I[u]|=|I[u^t]/t |<= 2Pi*log|1+u^t|/t .
>Como 0I[u]=0  nesse caso.
>Usando que I[u]=I[-u], vc conclui que I[u]=0 se |u|<= 1(em 1 eh obvio)
>
>Por outro lado, se |z|>1, trocando u por 1/u na definicao de I vem: 
>I(1/u)=Int[log(1-2cosx/u +1/u^2)] = 
>Int[log(u^2 - 2ucosx + 1)] - Int[2log|u|]
>Fazendo z = 1/u, temos que se |z|>1, entao |u|<1 e portanto:
>I(z) = 0 -2Pilog|u| = 2Pi*log|z|, o q conclui o problema..
>
>Marcio.
>
>
>-- Mensagem Original --
>De: Bruno Fernandes Cerqueira Leite <[EMAIL PROTECTED]>
>Para: [EMAIL PROTECTED]
>Enviar: 08:04 AM
>Assunto: Re: OBM-u
>
>At 00:30 23/10/01 -0200, you wrote:
>>Oi Bruno! Td bom? Tb achei a prova legal.. Qto ao resultado,
>acho que
>>fiz a 1 e a 5, 
>
>Como é que faz a 5? Eu tentei integral dupla, mas desisti logo...
>
>Bruno
>
>
>
>
>
>




Re: OBM-u

2001-10-23 Por tôpico Bruno Fernandes Cerqueira Leite

At 00:30 23/10/01 -0200, you wrote:
>Oi Bruno! Td bom? Tb achei a prova legal.. Qto ao resultado, acho que
>fiz a 1 e a 5, 

Como é que faz a 5? Eu tentei integral dupla, mas desisti logo...

Bruno

nao completei direito a 2 pq nao lembrava exatamente do
>enunciado (ou prova) de um teorema que tinha na Eureka 3 (no artigo de
>fracoes continuas) que me ajudaria muito. Na 4, que eu achei uma questao bem
>interessante, eu tmb
>escrevi.
>Fiquei um tempao, umas 2h30m ou mais escrevendo nela, mas ja descobri q
>errei uma bobagem na solucao.. Na hora achei logo uma recorrencia que
>parecia facilitar a coisa e acreditei nela.. Ela se mostrou util, mas eh
>provavelmente uma maneira bem horrivel de se fazer a questao.. Acabou
>demorando bem mais do que eu imaginava ...
>E voce, como foi na prova? Quais voce conseguiu fazer? Como foi o
>pessoal ai na USP? Do pessoal que eu conversei que fez a prova comigo,
>parece que a maioria foi mais ou menos igual a mim, acertando umas duas (O
>pessoal acertava em geral a 1 e a 4).
>Alem da 3, ainda nao consegui enxergar nada muito interessante na 6.
> Depois me mostraram algumas solucoes bem mais legais pra dois (a melhor
>que eu vi ateh agora criava 2 sequencias auxiliares para trelicas
>semelhantes a do problema e ai ficava bem simples.. )
>Bom, depois de reescrever tudo aqui no papel, ver onde eu tinha me enganado,
>e achar a nova resposta, nao aguentei e digitei aqui pra mandar pra lista
>tmb! Espero que vc e mais alguem alguem tenha paciencia de ler e/ou
>comentar! :)
>Minhas ideias na dois seguem no proximo email!
>
>Gostaria ainda de deixar uma pergunta sobre a questao 5. Eu consegui
>fazer a letra (b) usando a (a), mas hoje o
>Luciano me disse que tinha uma solucao legal do Nicolau integrando no plano
>complexo, que eu acabei esquecendo de perguntar.. Alguem (ou o proprio
>Nicolau) pode me mostrar como? Na prova, antes da (a), eu cheguei a tentar
>olhar pra integral como uma integral complexa no semicirculo de raio
>1(substituindo cosx = z + 1/z), mas nao sabia como achar os polos daquela
>funcao para poder integrar..
>
>Marcio
>
>- Original Message -
>From: "Bruno Fernandes Cerqueira Leite" <[EMAIL PROTECTED]>
>To: <[EMAIL PROTECTED]>
>Sent: Monday, October 22, 2001 10:52 PM
>Subject: Re: OBM
>> ...
>> Acho que a Nelly também cometeu erros tipográficos na questão 3 do nível
>> universitário! :-)
>>
>> Essa questão era beem difícil, eu até agora não sei como fazer um
>> avanço não trivial.
>>
>> Aliás, ninguém parabenizou ainda a prova da universitária, então eu vou
>> parabenizar: estava muito boa, as questões eram muito bonitas: valeu a
>pena
>> pensar 9 horas nelas! (eu até pensaria mais) Como vcs foram?
>>
>> Bruno Leite
>>
>> >[]s, N.
>> >
>> >
>>
>
>




Re: OBM-u

2001-10-23 Por tôpico Bruno Fernandes Cerqueira Leite

At 01:11 23/10/01 -0200, you wrote:
>   E aí, Márcio ! Pô, como eu já tinha falado contigo antes, qd cheguei em
>casa fiz de um jeito bem parecido com o seu, na força bruta mesmo. Mas na
>hora da prova eu fiz usando 2 funções, pra ver se montava uma recorrência e
>montei :) O problema é que eu errei em um pedacinho, aí os erros de conta
>foram carregados até o final... é uma pena...


>   Ah, eu queria saber se alguém poderia dar uma idéia pra 6. Eu cheguei a
>tentar um pouco na prova, e tentei mostrar algumsa coisas. Primeiro eu vi
>que o bordo ficava fixo. Mas não consegui provar que o centro era fixo, o
>que dificultou muito..

O centro dista menos de 1 de qualquer ponto, então f(centro) deve distar
menos de 1 de qualquer ponto. Como f é sobrejetora, se f(centro)!=centro,
teria um ponto tal que distancia(f(p)-f(centro))>1, absurdo.

O que eu notei que me ajudou muito é que a imagem inversa de dois pontos
diametralmente opostos deve ser dois pontos diam. opostos. Entao, por
exemplo, dá para concluir não só que f(bordo)=bordo, como essa f é uma
composição de rotações com espelhamento!

Aí eu provei que se x está entre O (centro) e P (na borda) então f(x) está
entre f(O) (centro) e f(P) (na borda) e ainda, a distância ao centro é
conservada por f.
( não foi bem isso que eu provei, mas eu não me lembro mais direito da
ordem das coisas - a minha solução usou 9 lemas...)

Aí, acho que fica fácil!

Bruno Leite

. Quando eu olhei pra essa questão, achei que tinha a
>ver com o teorema dos pontos fixos das contrações... é, aquilo era uma
>contraçào somente quando valia a desigualdade estrita. Daí, eu supus por
>"Ultra contradição" que valia a desigualdade estrita para todos e a partir
>daí tentei ver para quais pontos isso era imopssível ( queria concluir que
>não era possível para nenhum, né ). Para o bordo é óbvio... daí, pelo
>teorema dos pontos fixos das contrações, existe um único ponto no disco D,
>tal que f(a)=a, ou seja, apenas um ponto ficaria parado.

Uma rotação cumpre todas as condições do enunciado e só o centro fica
parado, então vc não poderia ter concluido que mais de um ponto fica parado.

 Mas aí não consegui
>formalizar minha idéia a partir daí.
> Abraços
>   Villard
>-Mensagem original-
>De: Marcio <[EMAIL PROTECTED]>
>Para: [EMAIL PROTECTED] <[EMAIL PROTECTED]>
>Data: Terça-feira, 23 de Outubro de 2001 00:57
>Assunto: Re: OBM-u
>
>
>>Oi Bruno! Td bom? Tb achei a prova legal.. Qto ao resultado, acho que
>>fiz a 1 e a 5, nao completei direito a 2 pq nao lembrava exatamente do
>>enunciado (ou prova) de um teorema que tinha na Eureka 3 (no artigo de
>>fracoes continuas) que me ajudaria muito. Na 4, que eu achei uma questao
>bem
>>interessante, eu tmb
>>escrevi.
>>Fiquei um tempao, umas 2h30m ou mais escrevendo nela, mas ja descobri q
>>errei uma bobagem na solucao.. Na hora achei logo uma recorrencia que
>>parecia facilitar a coisa e acreditei nela.. Ela se mostrou util, mas eh
>>provavelmente uma maneira bem horrivel de se fazer a questao.. Acabou
>>demorando bem mais do que eu imaginava ...
>>E voce, como foi na prova? Quais voce conseguiu fazer? Como foi o
>>pessoal ai na USP? Do pessoal que eu conversei que fez a prova comigo,
>>parece que a maioria foi mais ou menos igual a mim, acertando umas duas (O
>>pessoal acertava em geral a 1 e a 4).
>>Alem da 3, ainda nao consegui enxergar nada muito interessante na 6.
>> Depois me mostraram algumas solucoes bem mais legais pra dois (a
>melhor
>>que eu vi ateh agora criava 2 sequencias auxiliares para trelicas
>>semelhantes a do problema e ai ficava bem simples.. )
>>Bom, depois de reescrever tudo aqui no papel, ver onde eu tinha me
>enganado,
>>e achar a nova resposta, nao aguentei e digitei aqui pra mandar pra lista
>>tmb! Espero que vc e mais alguem alguem tenha paciencia de ler e/ou
>>comentar! :)
>>Minhas ideias na dois seguem no proximo email!
>>
>>Gostaria ainda de deixar uma pergunta sobre a questao 5. Eu consegui
>>fazer a letra (b) usando a (a), mas hoje o
>>Luciano me disse que tinha uma solucao legal do Nicolau integrando no plano
>>complexo, que eu acabei esquecendo de perguntar.. Alguem (ou o proprio
>>Nicolau) pode me mostrar como? Na prova, antes da (a), eu cheguei a tentar
>>olhar pra integral como uma integral complexa no semicirculo de raio
>>1(substituindo cosx = z + 1/z), mas nao sabia como achar os polos daquela
>>funcao para poder integrar..
>>
>>Marcio
>>
>>- Original Message -
>>From: "Bruno Fernandes Cerqueira Leite" <[EMAIL PROTECTED]>
>>To: <[EMAIL PROTECTED]>
>>Sent: Monday, October 22, 2001 10:52 PM
>>S

Re: OBM-u

2001-10-23 Por tôpico Bruno Fernandes Cerqueira Leite

At 00:30 23/10/01 -0200, you wrote:
>Oi Bruno! Td bom? Tb achei a prova legal.. Qto ao resultado, acho que
>fiz a 1 e a 5, nao completei direito a 2 pq nao lembrava exatamente do
>enunciado (ou prova) de um teorema que tinha na Eureka 3 (no artigo de
>fracoes continuas) que me ajudaria muito. Na 4, que eu achei uma questao bem
>interessante, eu tmb
>escrevi.

Podia usar o teorema da equidistribuição de {an} (a irracional, n natural)
mod 1 na questão 2?
Acho que se pudesse usar a questão ficaria quase trivial! (eu, por via das
dúvidas, não usei)

O teorema acima diz o seguinte (informal): a probabilidade de vc ter
x<{an}Fiquei um tempao, umas 2h30m ou mais escrevendo nela, mas ja descobri q
>errei uma bobagem na solucao.. Na hora achei logo uma recorrencia que
>parecia facilitar a coisa e acreditei nela.. Ela se mostrou util, mas eh
>provavelmente uma maneira bem horrivel de se fazer a questao.. Acabou
>demorando bem mais do que eu imaginava ...
>E voce, como foi na prova? Quais voce conseguiu fazer? Como foi o
>pessoal ai na USP? Do pessoal que eu conversei que fez a prova comigo,
>parece que a maioria foi mais ou menos igual a mim, acertando umas duas (O
>pessoal acertava em geral a 1 e a 4).
>Alem da 3, ainda nao consegui enxergar nada muito interessante na 6.
> Depois me mostraram algumas solucoes bem mais legais pra dois (a melhor
>que eu vi ateh agora criava 2 sequencias auxiliares para trelicas
>semelhantes a do problema e ai ficava bem simples.. )

Aí vai minha solução para a dois:

Seja y=ax a reta. Se a é racional é trivial. Então suponha a
irracional.Basta provarmos que a equação em n, 

{an}Bom, depois de reescrever tudo aqui no papel, ver onde eu tinha me enganado,
>e achar a nova resposta, nao aguentei e digitei aqui pra mandar pra lista
>tmb! Espero que vc e mais alguem alguem tenha paciencia de ler e/ou
>comentar! :)
>Minhas ideias na dois seguem no proximo email!
>
>Gostaria ainda de deixar uma pergunta sobre a questao 5. Eu consegui
>fazer a letra (b) usando a (a), mas hoje o
>Luciano me disse que tinha uma solucao legal do Nicolau integrando no plano
>complexo, que eu acabei esquecendo de perguntar.. Alguem (ou o proprio
>Nicolau) pode me mostrar como? Na prova, antes da (a), eu cheguei a tentar
>olhar pra integral como uma integral complexa no semicirculo de raio
>1(substituindo cosx = z + 1/z), mas nao sabia como achar os polos daquela
>funcao para poder integrar..
>
>Marcio
>
>- Original Message -
>From: "Bruno Fernandes Cerqueira Leite" <[EMAIL PROTECTED]>
>To: <[EMAIL PROTECTED]>
>Sent: Monday, October 22, 2001 10:52 PM
>Subject: Re: OBM
>> ...
>> Acho que a Nelly também cometeu erros tipográficos na questão 3 do nível
>> universitário! :-)
>>
>> Essa questão era beem difícil, eu até agora não sei como fazer um
>> avanço não trivial.
>>
>> Aliás, ninguém parabenizou ainda a prova da universitária, então eu vou
>> parabenizar: estava muito boa, as questões eram muito bonitas: valeu a
>pena
>> pensar 9 horas nelas! (eu até pensaria mais) Como vcs foram?
>>
>> Bruno Leite
>>
>> >[]s, N.
>> >
>> >
>>
>
>




Re: OBM

2001-10-22 Por tôpico Bruno Fernandes Cerqueira Leite

(...)
>
>Acho que já posso contar um segredo: esta questão foi gerada por um erro
>tipográfico da Nelly quando ela transcrevia uma questão bem mais fácil.
>Ficou ótima assim. :-)


Acho que a Nelly também cometeu erros tipográficos na questão 3 do nível
universitário! :-)

Essa questão era beem difícil, eu até agora não sei como fazer um
avanço não trivial.

Aliás, ninguém parabenizou ainda a prova da universitária, então eu vou
parabenizar: estava muito boa, as questões eram muito bonitas: valeu a pena
pensar 9 horas nelas! (eu até pensaria mais) Como vcs foram?

Bruno Leite

>[]s, N.
>
>




Re: Construção GeométricaXRyaWNh

2001-10-19 Por tôpico Bruno Fernandes Cerqueira Leite

At 21:13 18/10/01 -0300, you wrote:
>Saudações,
>
>Gostaria q alguém me ajudasse na solução do seguinte problema:
>
>Sejam P e Q os pontos de interseção de dois círculos. Construir o segmento
>APB, tal que AP=PB.(A pertence a um círculo e B ao outro).

Não basta tomar a reta perpendiclar a PQ passando por P? (A e B serão as
interseções destas retas com os círculos)

Bruno Leite


>Agradeço.
>
>
>Araújo.
>
>




Re: Função inversaZlcnNh

2001-09-28 Por tôpico Bruno Fernandes Cerqueira Leite

At 19:24 27/09/01 -0300, you wrote:
>Sauda,c~oes,
>
>Não posso ajudá-lo. Meus conhecimentos
>nesta área são muito mais informativos (só
>tenho a informação) do que "formativos" (a
>explicação).
>
>Mas os manuais de fórmulas e tabelas costumam
>falar alguma coisa a respeito.
>
>De qualquer jeito o assunto é avançado e uma
>simples definição seria de pouca utilidade.
>
>Alguém poderia dar uma referência que mostra
>a solução para os polinômios do 5o.grau? Sem
>necessariamente seguir o estudo de Galois. Acho
>que já li um livro que tratava o assunto desta maneira.

O 25º problema em "100 great problems of mathematics - Their History and
solution", de Heinrich Dorrie, ed. Dover,
trata exatamente disto.

Aliás este livro tem várias outras coisas interessantes em álgebra,
geometria e teoria dos números.(ex: teorema fundamental da álgebra,
reciprocidade quadrática, último teorema de fermat no caso n=3, construções
com régua e compasso)

Bruno Leite
www.ime.usp.br/~brleite

>
>[]'s
>Luís
>
>-Mensagem Original-
>De: <[EMAIL PROTECTED]>
>Para: <[EMAIL PROTECTED]>
>Enviada em: Terça-feira, 25 de Setembro de 2001 17:46
>Assunto: Re: Função inversa
>
>
>Por favor, defina função elíptica...em livros de história da matemática são
>amplamente citadas, mas não definidas
>Um abraço
>Crom
>
>




Re: Função inversaZlcnNh

2001-09-20 Por tôpico Bruno Fernandes Cerqueira Leite

At 14:04 20/09/01 -0300, you wrote:
>
>Realmente sua funcao admite inversa, pois ela eh uma bijecao crescente dos 
>reais nos reais. Para provar isso, vc pode ver que a derivada primeira de f 
>eh 5x^4 + 1 > 0 para todo x, o que garante que f eh estritamente crescente. 
>Por outro lado, dado qualquer y real existe um x real tal que x^5 + x+1 = y, 
>pois todo polinomio de grau impar possui ao menos uma raiz real. O polinomio 
>aqui eh x^5 + x + 1-y = 0 (*). O problema eh que voce nao pode explicitar x 
>em termos de y na equacao (*) por meio de uma formula. Isso porque nao 
>existe formula que deh as raizes de uma equacao do quinto grau em termos de 
>seus coeficientes (a prova desse fato nao estah no nivel do ensino medio). 

Mas isso não seria só para a equação _geral_do 5º grau? Talvez haja um
truquezinho aí que nos pemita isolar o x em função do y nesta equação
específica (eu duvido um pouco mas não vejo motivo para que isso não ocorra)

Por exemplo se o problema fosse achar a inversa de x^5 +1 é evidente que é
possível isolar o x.

Não sei se fui muito claro... (e nem sei se falei besteira)

Bruno Leite


>Assim, o maximo que vc pode fazer eh saber que a inversa exista, mas nao 
>pode explicita-la.
>
>
>
>>estou tentando obter a inversa da seguinte função:
>>
>>f(X)=x^5 +x +1
>>
>>Alguem poderia me ajudar
>
>
>_
>Chegou o novo MSN Explorer. Instale já. É gratuito! 
>http://explorer.msn.com.br
>
>




Re: Probleminhas

2001-09-19 Por tôpico Bruno Fernandes Cerqueira Leite

At 03:29 19/09/01 +, you wrote:
>Alguem pode mandar a soluçao dos problemas?
>
>1.Mostre que
>n
>SUM (1/j) não é um número inteiro.
>j=1

Se vc consegue ver arquivos PS ou DVI, veja

http://www.ime.usp.br/~brleite/arquivos/MatConcr.ps
ou 
http://www.ime.usp.br/~brleite/arquivos/MatConcr.dvi

Há duas soluções deste problema lá.

>
>2. Mostre que todo o primo da forma 3k+1 tb é da forma 6k+1.
>valeu

Um número 3k+1 ou é 6m+1 ou 6m+4 pois deixa resto 1 na divisão por 3. Mas
6m+4 nunca é primo, ao menos se 6m+4>0 (se m=-1, 6m+4 é primo mas acho que
vc está supondo tudo positivo)

Bruno Leite
www.ime.usp.br/~brleite


>[]'s, M.
>
>_
>Get your FREE download of MSN Explorer at http://explorer.msn.com/intl.asp
>
>




Re: ajuda

2001-09-09 Por tôpico Bruno Fernandes Cerqueira Leite

de novo este problema??

Bruno


At 07:50 09/09/01 EDT, [EMAIL PROTECTED] wrote:
>"Ao estudar as ciências, os exercícios são mais úteis do que as regras" 
> Assim escreveu Isaac Newton em sua aritmética Universal e, de fato,  
>acompanhava as indicações teóricas com uma série de exemplos. No meio
deles,  
>acha-se o problema das vacas que pastam em uma campina, exemplo esse que
deu  
>origem a um tipo específico de problemas semelhantes a ele. 
> 
> O capim cresce no pasto todo com igual velocidade e espessura. Sabe-se  
>que 70 vacas o comeriam em 24 dias; 30 vacas em 60 dias. Quantas vacas  
>comeriam todo o capim em 96 dias? 
> Duas pessoas, tentando resolver este problema, esforçavam-se por  
>descobrir sua resposta. 
> - Que resultado esquisito! - disse um deles. 
> - Se em 24 dias, 70 vacas comem o capim todo, quantas comeriam em 96  
>dias? Está visto que é 1/4 de 70, isto é, 17,5 vacas... Este é o primeiro  
>absurdo! o segundo, mais esquisito ainda, é que se 30 vacas comem o capim
em  
>60 dias, em 96 dias o capim será comido por 18,75 vacas. E ainda por cima,
se  
>70 vacas comem em 24 dias, 30 gastarão nisso 56 dias e não 60, como afirma
o  
>problema. 
> - Mas você está levando em conta que o capim cresce sem parar? -  
>perguntou o outro. 
> A observação procedia. A grama cresce sem cessar, fato esse que não  
>pode ser esquecido, pois que, se o fizéssemos, não só o problema ficaria
sem  
>solução, mas as suas próprias condições pareceriam contraditórias. 
> Podemos então concluir que 21 vacas comeriam o capim em quantos dias? 
> 
>Aguardo comentários. Grato!! 




Re: Ajuda para achar um livro

2001-09-06 Por tôpico Bruno Fernandes Cerqueira Leite

At 18:06 06/09/01 -0300, you wrote:
>Sauda,c~oes,
>
>Será que alguém teria alguma coisa
>a dizer?
>
>Um abraço,
>Luís
>
>> -- Forwarded Message
>> From: "rodferro"<[EMAIL PROTECTED]>
>> Date: Wed,  5 Sep 2001 00:17:24 -0300
>> To: [EMAIL PROTECTED]
>> Subject: duvida-
>>
>> Caro professor,
>> estava lendo o livro Manual de progressoes,(que eh muito
>> bom ,parabens),quando eu deparei na bibliografia o
>> seguinte livro:
>> Shlarsky,D.O.,Chentzov,N.N,and Yaglom,I.M,The USSR
>> Olympiad Problem Book,W.H Freeman,1962.
>> Estou procurando por esse livro faz tempo,ja procurei em
>> sebo ,em varias bibliotecas,enfim muitos lugares.O
>> senhor saberia me informar onde posso compra-lo,ou
>> consulta-lo.(Pode ser tanto no rio ou em sp).
>> Muito Obrigado.
>> Rodrigo.

Na Amazon certamente tem, e na livraria cultura (www.livcultura.com.br)
eles devem aceitar encomendas.

Bruno

>> (Parabens mais uma vez pelo livro).
>> -- End of Forwarded Message
>>
>
>




Re: 2 QUESTÕES

2001-08-28 Por tôpico Bruno Fernandes Cerqueira Leite

Oi Henrique

Suponha que não tenhamos x=y=z. Então temos, sem perda de generalidade,
x>=z e x>y. ( x é o maior dos três)
Como 2x/(1+x)=z<=x, temos x>=1.

Por outro lado, como 2y/(1+y)=x>y, temos y<1. Mas então 2y<1+y e
2y/(1+y)<1, absurdo pois isso implica que x<1.

Bruno Leite
www.ime.usp.br/~brleite


At 01:42 29/08/01 +, you wrote:
>OLÁ,
>Gostaria da ajuda de vcs nas seguintes questões:
>1.Os numeros positivos x,y e z são tais que:
>x=2y/1+y , y=2z/1+z e z=2x/1+x.
>prove q x=y=z
>2. Determine todas as funções estritamente crescentes f:N*->N* tais que 
>f(n+f(n)=2f(n)
>  valeu!
>
>
>
>_
>Get your FREE download of MSN Explorer at http://explorer.msn.com/intl.asp
>
>




Re: Combinatória e Eq. 3 grau

2001-08-25 Por tôpico Bruno Fernandes Cerqueira Leite

At 22:49 24/08/01 -0300, you wrote:
>Oi.   Bom, vamos chamar de t(n) o  numero de vezes em q mexeram no
>armario n. Mais explicitamente, t(n) é qtos divisores inteiros
>positivos tem  n. É claro q o armário n estará aberto
>ao final sse t(n) for ímpar"" se anulam). Entao só precisamos
>calcular t(n).   Para isso, sendo [(p_1)^k_1]*[(p_2)^k_2]*...[(p_r)^k_r] a 
>fatoracao prima de n, é<<=k_i. Entao teremos
>[(k_1)+1]*[(k_2)+1]*...*[(k_r)+1]  possibilidades para os divisores de n,
>i.e., t(n) = [(k_1)+1]*[(k_2)+1]*...*[(k_r)+1], onde n = 
>[(p_1)^k_1]*[(p_2)^k_2]*...[(p_r)^k_r].   Logo, como queremos t(n) 
>ímpar, nenhum dos [(k_i)+1] pode ser par, ou seja, todos tem q ser 
>ímpares, ou seja, todos os k_i tem de ser pares. Mas então
>podemos  tomar k_i = 2*q_i, e n = 
>{[(p_1)^q_1]*[(p_2)^q_2]*...[(p_r)^q_r]}^2.   Só"prová"...  
>t+! 

Um outro modo de ver que o numero de divisores de um número é impar sse ele
é quadrado é o seguinte.

Para um número n que não é quadrado, se d é divisor de n, então n/d é
divisor de n. Então os divisores de n vem aos pares! Portanto, o número de
divisores de n será par.

Para um quadrado este argumento não vale pois para o divisor d=raiz(n), o
seu par n/d é a própria raiz(n), ou seja, um único cara não tem par.
Portanto aqui o número de divisores de n será ímpar.

Bruno Leite
http://www.ime.usp.br/~brleite

  -Mensagem original-
>De:  <[EMAIL PROTECTED]>
>Para:  [EMAIL PROTECTED]<[EMAIL PROTECTED]>
>Data:  Sexta-feira, 24 de Agosto de 2001 19:51
>Assunto:  Combinatória e Eq. 3 grau
>
> Olá, aí vai uma questão que jah  esteve aqui
>na lista mas para a qual eu ainda nao vi uma soluçao... 
>mostrei-a a meu professor e ele chegou à mesma conclusao que eu
>havia  chegado, no entanto, assim como eu, ele nao conseguiu demonstrar
>a  prová"".1. Em um corredor há 900
>armários, numerados  de 1 a 900, 
>inicialmente todos fechados. 900 pessoas, numeradas de 1  
>a 900, atravessam o corredor. A pessoa de número k 
>reverte o  estado de todos os armários cujos números
>sâo  
>múltiplos de k. Por exemplo, a pessoa de número 4 mexe  
>nos armários de números 4, 8, 12,..., abrindo os que  
>encontra fechados e fechando os que encontra abertos. ao 
>final,  quais armários ficarão abertos?
> serah q alguém podia mostrar uma  solução??? 
> ah, aí vai uma duvida bem trivial, quais sao as 
>raízes da equação x^3 -4x -1 = 0 ? Como eu faço
> para encontrá-las?   abraços Hugo
>